Jump to content

sinh vien's Content

There have been 261 items by sinh vien (Search limited from 24-05-2020)



Sort by                Order  

#554976 Tuyển tập đề thi putnam full

Posted by sinh vien on 19-04-2015 - 08:10 in Thảo luận về các kì thi, các kì kiểm tra Toán sinh viên

Đây là tài liệu sưu tầm của mình ( các bài toán đều có lời giải hết) :

putnam 1938 1964

Attached File  Putnam 1938-1964 (2) (1).pdf   30.4MB   283 downloads

putnam 1965 1984Attached File  Putnam 1965-1984.pdf   8.12MB   644 downloads

putnam 1985 2000 Attached File  Putnam1985-2000.pdf   2.06MB   253 downloads

putnam 2001 bài toán Attached File  2001.pdf   77.83KB   238 downloads lời giảiAttached File  2001s.pdf   112.51KB   380 downloads

putnam 2002  bài toánAttached File  Undergraduate_Competitions-Putnam-2002-23.pdf   141.34KB   272 downloads lời giải Attached File  2002s.pdf   101.25KB   217 downloads

putnam 2003  bài toán Attached File  Undergraduate_Competitions-Putnam-2003-23.pdf   129.57KB   234 downloads lời giảiAttached File  2003s.pdf   87.94KB   402 downloadsAttached File  2003s.pdf   87.94KB   402 downloads

putnam 2004 bài toán Attached File  Undergraduate_Competitions-Putnam-2004-23.pdf   149.68KB   196 downloads lờ giảiAttached File  2004s.pdf   71.13KB   189 downloads

putnam 2005 bài toán Attached File  Undergraduate_Competitions-Putnam-2005-23.pdf   142.35KB   247 downloadsAttached File  Undergraduate_Competitions-Putnam-2003-23.pdf   129.57KB   234 downloads lời giải

putnam 2006 Attached File  2006.pdf   208KB   599 downloads

putnam 2007 bài toán Attached File  Undergraduate_Competitions-Putnam-2007-23.pdf   137.65KB   245 downloads lời giải Attached File  2007s.pdf   81.36KB   266 downloads

putnam 2008  bài toán Attached File  Undergraduate_Competitions-Putnam-2008-23.pdf   130.43KB   230 downloads lời giải

 Attached File  Putnam_2008 (1).pdf   141.76KB   598 downloads

putnam 2009 bài toán Attached File  Undergraduate_Competitions-Putnam-2009-23.pdf   132.36KB   237 downloads lời giảiAttached File  Putnam_2009.pdf   200.84KB   907 downloads

putnam 2010 bài  toán Attached File  Undergraduate_Competitions-Putnam-2010-23.pdf   142.71KB   228 downloads lời giải 

putnam 2011 bài toán Attached File  Undergraduate_Competitions-Putnam-2011-23.pdf   141.02KB   259 downloads lời giải

Attached File  2011s.pdf   76.89KB   220 downloads

putnam 2012Attached File  sol2012.pdf   87.86KB   209 downloads

putnam 2013Attached File  sol2013.pdf   228.77KB   217 downloads

putnam 2014 bài toán Attached File  4319.pdf   117.51KB   352 downloads lời giải Attached File  putnam2014.pdf   150.09KB   327 downloads

 Mong rằng đây sẽ là bộ tài liêu hữu ích cho đông đảo mọi người

Attached Files




#554984 Tuyển tập đề thi putnam full

Posted by sinh vien on 19-04-2015 - 08:35 in Thảo luận về các kì thi, các kì kiểm tra Toán sinh viên

Không có bạn ơi . Tài liệu này chỉ có bản tiếng anh à !




#555030 Tài liệu gồm 4 tập sách của Ramanujan

Posted by sinh vien on 19-04-2015 - 12:05 in Tài liệu, chuyên đề Toán cao cấp

Đây là một bộ tài liệu rất hữu ích cho những ai đam mê giải tích

Quyển 1 Attached File  Ramanujan's Notebooks I.pdf   12.41MB   971 downloads

Quyển 2 Attached File  Ramanujan's Notebooks II.pdf   13.64MB   406 downloads

Quyển 3 Attached File  Ramanujan's Notebooks III.pdf   20.03MB   405 downloads 

Quyển 4 Attached File  Ramanujan's Notebooks IV.pdf   10.63MB   335 downloads

 còn đây là  cuốn sách lý thuyết số - giải tích tổng hợp cô động các thành tựu của ramanujan

 Attached File  Bruce C. Berndt - Number Theory in the Spirit of Ramanujan - AMS - 201p.pdf   14.02MB   501 downloads




#555191 Tuyển tập đề thi Baltic Way

Posted by sinh vien on 20-04-2015 - 08:44 in Tài liệu tham khảo khác

Sau bộ tài liệu putnam đây là một bộ tài liệu dành cho phổ thông tốt :

1990 -Attached File  bw90sol.pdf   61.88KB   136 downloads

1991Attached File  bw91sol.pdf   69.69KB   155 downloads

1992Attached File  bw92sol.pdf   73.12KB   136 downloads

1993Attached File  bw93sol.pdf   80.08KB   156 downloads

1994Attached File  bw94sol.pdf   87.38KB   129 downloads

1995Attached File  bw95sol.pdf   73.61KB   148 downloads

1996Attached File  bw96sol.pdf   81.36KB   111 downloads

1997Attached File  bw97sol.pdf   123.31KB   104 downloads

1998Attached File  bw98sol.pdf   121.46KB   109 downloads

1999Attached File  bw99sol.pdf   124.71KB   110 downloads

2000Attached File  bw00sol.pdf   102.75KB   107 downloads

2001Attached File  bw01sol.pdf   95.83KB   118 downloads

2002Attached File  bw02sol.pdf   121.4KB   129 downloads

2003Attached File  bw03sol.pdf   157.74KB   151 downloads

2004Attached File  bw04sol.pdf   140.7KB   114 downloads

2005Attached File  bw05sol.pdf   144.51KB   168 downloads

2006Attached File  bw06sol.pdf   145.27KB   106 downloads

2007Attached File  bw07sol.pdf   207.33KB   107 downloads

2008Attached File  bw08sol.pdf   159.89KB   109 downloads

2009Attached File  bw09pb.pdf   49.85KB   118 downloads (năm này không thấy có lời giải cụ thể các bạn có thể tham khảo trên trang '' art problem solving'' các lời giải của từng bài )

2010Attached File  bw10sol.pdf   224.59KB   161 downloads

2011Attached File  bw11sol.pdf   352.37KB   146 downloads

2012Attached File  bw12sol.pdf   204.93KB   261 downloads

2013Attached File  PROBSOLS.pdf   1.29MB   240 downloads

     Mong rằng với số lượng tài liệu ít ỏi này sẽ giúp ít cho nhiều bạn




#555192 Tài liệu đề thi thạc sĩ của Rumani

Posted by sinh vien on 20-04-2015 - 09:02 in Tài liệu, chuyên đề Toán cao cấp

 Sau hai bộ '' full' nam và ' baltic way ' thì đây là bộ tài liệu khó và còn bị ít mong các bạn thông cảm

   lần 2Attached File  solutions2009.pdf   99.41KB   173 downloads

  lân 4 ngày 1Attached File  Sols2011D1.pdf   62.76KB   111 downloads ngày 2 Attached File  Sols2011D2.pdf   99.56KB   104 downloads

lần 5 ngày 1 Attached File  Solutions2012-1.pdf   60.69KB   107 downloads ngày 2Attached File  Solutions2012-2.pdf   67.87KB   105 downloads

lần 6 ngày 1Attached File  Solutions2013-1.pdf   188.4KB   209 downloads ngày 2 Attached File  Solutions2013-2.pdf   183.82KB   118 downloads

lân 7 ngày 1Attached File  Solutions_RMM2015-1.pdf   166.11KB   98 downloads ngày 2 Attached File  Solutions_RMM2015-2.pdf   173.35KB   97 downloads

  Nếu có bạn nào có các lần thi bị khuyết xin hãy chia sẻ ở đây nhé !




#555380 tuyển chọn các bài toán tính định thức

Posted by sinh vien on 21-04-2015 - 08:53 in Đại số tuyến tính, Hình học giải tích

Sau đây là một số bài toán tính định thức theo các hướng độc đáo đã được sau trên các cuộc thi danh tiếng  (đề mục này sẽ được bổ sung dần dần )

   

Bài toán 1. (PUTNAM 2014)  Cho A là ma trận $n\times n$ trong đó phần tử ở hàng thứ i cột thứ j được cho bởi $\frac{1}{min(i,j)}$ với $1\leqslant i,j\leqslant n$ . Tính định thức của ma trận A

Lời giải. Từ điều kiện ở đầu ta thấy 

           A=$\begin{bmatrix} 1 & 1 &1 ...& 1& \\ 1& \frac{1}{2} &\frac{1}{2} ...& \frac{1}{2}& \\ 1 & \frac{1}{2}&\frac{1}{3} ...&\frac{1}{3} & \\1 &\frac{1}{2}&\frac{1}{3}...& \frac{1}{4}\\ ... & ... & ... &... \\ 1 & \frac{1}{2}&\frac{1}{3} ... & \frac{1}{n} & \end{bmatrix}$

   Để giải bài toán này ta sẽ áp dụng khai triển Laplace  cho hàng thứ n . Chú ý khi đó ma trận con thu được khi xóa hàng thứ n có dạng 

         \begin{bmatrix} 1 & 1& 1 & ... &1& 1\\1 & \frac{1}{2}&\frac{1}{2} & ...& \frac{1}{2} & \frac{1}{2}\\ ... & ... &... & ...&... & ...\\1 & \frac{1}{2}&\frac{1}{3} &..& \frac{1}{n-1} &\frac{1}{n-1} \end{bmatrix}$

Khi đó cột thứ n và cột thứ n-1 trùng nhau nên ma trận con chứa cả hai cột n và n-1 có định thức bằng 0 . Từ đó ta thấy

        $\begin{vmatrix} 1& 1 & 1 & ... &1 \\1 & \frac{1}{2} & \frac{1}{2} &... & \frac{1}{2}\\ 1& \frac{1}{2}&\frac{1}{2} &... &\frac{1}{2} \\ ...& ... &... & ... &... \\ 1& \frac{1}{2} &\frac{1}{3} & ... &\frac{1}{n} \end{vmatrix}=-\frac{1}{n-1}\begin{vmatrix} 1 & 1 & ... &1 \\ 1 & \frac{1}{2} &... &\frac{1}{2} \\... & ...& ... & ...\\ 1 & \frac{1}{2}&... &\frac{1}{n-1} \end{vmatrix}+\frac{1}{n}\begin{vmatrix} 1 & 1 & ... & 1\\1 & \frac{1}{2} & ... & \frac{1}{2}\\ ... &\... & ... &... \\ 1 & \frac{1}{2} &... &\frac{1}{n-1} \end{vmatrix}$

    Nếu đặt $D_{n}=det(A)$ trong đó A có cấp n thì $D_{n}=\left ( \frac{1}{n} -\frac{1}{n-1}\right ) D_{n}=\frac{-1}{n(n-1)}D_{n-1}$

    Sử dụng hệ thức truy hồi này và chú ý $D_{1}=1$ ta được

$D_{n}=\frac{-1}{n(n-1)}\frac{-1}{(n-1)(n-2)}...\frac{-1}{3.2}\frac{-1}{2.1}=\frac{(-1)^{n+1}}{n!(n-1)!}$

   Bài toán 2 (ĐHBK -2013) .Cho $x_{i},y_{i},1\leq i\leq n$ là các số phức với $x_{i}y_{j}$$\neq 1$ với mọi cặp $x_{i},y_{j}$.

  Tính định thức của ma trận $M=(m_{ij})_{n\times n}$ tróng đó $m_{ij}=\frac{1}{1-x_{i}y_{j}}$

lời giải. Để cho thuận tiện ta quy ước $D_{y_{1}y_{2}...y_{n}}^{x_{1}x_{2}...x_{n}}=det(M)$. Ta thấy

 n=2 : $D_{y_{1}y_{2}}^{x_{1}x_{2}}=\begin{vmatrix} \frac{1}{1-x_{1}y_{1}} &\frac{1}{1-x_{1}y_{2}} \\ \frac{1}{1-x_{2}y_{1}} &\frac{1}{1-x_{2}y_{2}} \end{vmatrix}=\frac{(x_{1}-x_{2})(y_{1}-y_{2})}{(1-x_{1}y_{1})(1-x_{1}y_{2})(1-x_{2}y_{1})(1-x_{2}y_{2})}$  ( tính toán tương đối đơn giản nên mình không nêu ra cụ thể )

Ta có thể tính trực tiếp thêm một số giá trị của n . Dự đoán :

$D_{y_{1}...y_{n}}^{x_{1}...x_{n}}=\frac{\prod_{1\leqslant i< j\leqslant n}(x_{i}-x_{j})(y_{i}-y_{j})}{\prod_{1\leq i,j\leq n}(1-x_{i}y_{j})}$. Ta sẽ chứng minh quy nạp công thức này.

   Áp dụng khai triển Laplace cho cột thứ nhất ta được

$D_{y_{1}..y_{n+1}}^{x_{1}...y_{n+1}}=\sum_{i=1}^{n+1}(-1)^{i+1}\frac{1}{1-x_{i}y_{1}}D_{y_{2}...y_{n+1}}^{x_{1}...x_{i-1}x_{i+1}...x_{n+1}}$

$=\sum_{i=1}^{n+1}(-1)^{i-1}\frac{1}{1-x_{i}y_{1}}\frac{\prod_{1\leq k< l\leq n+1,k,l\neq i}(x_{k}-x_{l})\prod_{2\leq k< l\leq n+1}(y_{k}-y_{l})}{\prod_{1\leq k\leqslant n+1,k\neq i,2\leq l\leqslant n+1}(1-x_{k}y _{l})}$

$\sum_{i=1}^{n+1}(-1)^{i-1}\prod_{k=1,k\neq i}^{n+1}(1-x_{k}y_{1})\frac{\prod_{1\leqslant k< l\leq n+1}(x_{k}-x_{l})\prod_{k=2}^{n+1}(1-x_{i}y_{k})\prod_{2\leq k< l\leq n+1}(y_{k}-y_{l})}{\prod_{k=1}^{i-1}(x_{k}-x_{i})\prod_{k=i+1}^{n+1}(x_{i}-x_{k})\prod_{1\leqslant k,l\leqslant n+1}(1-x_{k}y_{l})}$

$=\left ( \sum_{i=1}^{n+1}\prod_{k=2}^{n+1}(1-x_{i}y_{k})\prod_{k=1,k\neq i}^{n+1} \frac{1-x_{k}y_{1}}{x_{i}-x_{k}}\right )\frac{\prod_{1\leq k< l\leq n+1}(x_{k}-x_{l})\prod_{2\leq k< l\leq n+1}(y_{k}-y_{l})}{\prod_{1\leqslant k,l\leq n+1}(1-x_{k}y_{l})}$

 Đặt $P(x)=\prod_{k=2}^{n+1}(1-xy_{k})$ thì $P(x)$ là một đa thức bậc n và $P(x_{i})=\prod_{k=2}^{n+1}(1-x_{i}y_{k}),i=1,2,..n+1$ . Áp dụng công thức Lagrange ta được:

  $P(x)=\sum_{i=1}^{n+1}\prod_{k=2}^{n+1}(1-x_{i}y_{k})\prod_{k=1,k\neq i}^{n+1}\frac{x-x_{k}}{x_{i}-x_{k}}$

 Thay $x=\frac{1}{y_{1}}$ , nhân hai vế cho $y_{1}^{n}$ ta được

      $\prod_{k=2}^{n+1}(y_{1}-y_{k})=\sum_{i=1}^{n+1}\prod_{k=2}^{n+1}(1-x_{i}y_{k})\prod_{k=1,k\neq i}^{n+1}\frac{1-x_{k}y_{1}}{x_{i}-x_{k}}$.

  Từ đây ta dễ dàng thu được kết quả mong muốn.




#555385 Sử dụng các công cụ đại số hiện đại giải quyết các bài toán tổ hợp

Posted by sinh vien on 21-04-2015 - 09:46 in Thảo luận về các kì thi, các kì kiểm tra Toán sinh viên

                           Chuyên đề này sẽ luôn được bổ sung mong các bạn ủng hộ

Bài toán 1 (PUTNAM 2005 ) Cho $S_{n}$ là tập các hoán vị của tập hợp $\left \{ 1,2,... n\right \}.$. Xét  $\pi \in S_{n}$ , đặt $\sigma (\pi )=1$ nếu $\pi$ là hoán vị chẵn và $\sigma (\pi )=-1$ nếu$\pi$ là hoán vị lẻ. Gọi $\nu (\pi )$ là số các điểm bất động của hoán vị $\pi$.

 Chứng minh đẳng thức :

                                                             $\sum_{\pi \in S_{n}}^{} \therefore \frac{\sigma (\pi )}{\nu (\pi )+1}=(-1)^{n+1}\frac{n}{n+1}$

Lời giải 

  Đặt $I$ là ma trận đơn vị cấp n , $J_{x}$ là ma trận có các phần tử trên đường chéo chính bằng $x$ và các phần tử còn lại bằng 1. Khi đó

                          $J_{x}=\begin{bmatrix} x & 1 & ... &1 \\ 1& x &... &1 \\... & ... &... & ...\\1 & 1 & ... &x \end{bmatrix}$

 Ta có thể dễ dàng tính được định thức của ma trận này $detJ_{x}=(x+n-1)(x-1)^{n-1}$ ( có thể tìm thấy trong nhiều tài liệu hiện nó nên không nêu ra ở đây)

  Mặc khác, chúng ta có thể tính tổng này theo tổng các hoán vị :

                       $detJ_{x}=\sum_{\pi \in S_{n}}sgn(\pi )x^{\upsilon (\pi )}$

 Lấy tích phân từ 0 tới 1 ( có sử dụng đến phép đổi biến $y=1-x$ ta thu được

  $\sum_{\pi \in S_{n}}\frac{sgn(\pi )}{\upsilon(\pi )+1}=\int_{0}^{1}(x+n-1)(x-1)^{n-1}dx=\int_{0}^{1}(-1)^{n+1}(n-y)y^{n-1}dy=(-1)^{n+1}\frac{n}{n+1}$

 Chú ý $\sigma (\pi )$ trong định nghĩa của bài toán chính là định nghĩa về dấu của phép thế nên ta có được đpcm




#555596 Tính tích phân dựa vào phương trình vi phân

Posted by sinh vien on 22-04-2015 - 09:20 in Giải tích

Bài toán .Tính tích phân :

                       $\int_{0}^{\infty }e^{-\frac{t^{2}}{2}}cos\frac{x^{2}}{2t^{2}}dt$

Lời giải:

Ta thấy hàm 

  $y(x)=\int_{0}^{\infty }e^{-\frac{t^{2}}{2}}cos\frac{x^{2}}{2t^{2}}dt$ thỏa mãn phương trình vi phân

$y^{(iv)}+y=0$. Thật vậy:

$y^{'}=\int_{0}^{\infty }e^{-\frac{t^{2}}{2}}sin\frac{x^{2}}{2t^{2}}\frac{-x}{t^{2}}dt=-\int_{0}^{\infty }e^{-\frac{x^{2}}{2u^{2}}}sin\frac{u^{2}}{2}du$

và $y^{''}=-\int_{0}^{\infty }e^{-\frac{x^{2}}{2u^{2}}}sin\frac{u^{2}}{2}\frac{-x}{u^{2}}du=\int_{0}^{\infty }e^{-\frac{t^{2}}{2}}sin\frac{x^{2}}{2t^{2}}dt$

 tiếp tục quá trình tính toán này ta sẽ thu được

   $y^{(iv)}=-\int_{0}^{\infty }e^{-\frac{t^{2}}{2}}cos\frac{x^{2}}{2t^{2}}dt$

các kết quả trên chứng minh nhận định của ta.

   Dựa vào lý thuyết tổng quát về phương trình vi phân tuyến tính hệ số hằng ta thấy, nghiệm tổng quát của phương trình vi phân đã dẫn là :

 $y(x)=e^{\frac{x}{\sqrt{2}}}(C_{1}cos\frac{x}{\sqrt{2}}+C_{2}sin\frac{x}{\sqrt{2}})+e^{-\frac{x}{\sqrt{2}}}(C_{3}cos\frac{x}{\sqrt{2}}+C_{4}sin\frac{x}{\sqrt{2}})$

  Để tính tích phân ở đầu bài ta đi tìm một nghiệm riêng của phương trình vi phân thỏa mãn $y(0)=\int_{0}^{\infty }e^{-\frac{t^{2}}{2}}dt=\sqrt{\frac{\pi }{2}}$ ( tích phân dạng Gauss) , $y^{'}(0)=-\int_{0}^{\infty }sin\frac{u^{2}}{2}du=-\frac{\sqrt{\pi }}{2}$ ( tích phân  dạng  Fresenel ),$y^{''}(0)=0$ và $y^{'''}(0)=\int_{0}^{\infty }cos\frac{u^{2}}{2}du=\frac{\sqrt{\pi }}{2}.$. Dựa vào các điều kiện ban đầu này ta dễ dàng tính được: $C_{1}=C_{2}=C_{3}=0$ và $C_{4}=\sqrt{\frac{\pi }{2}}$

 Do vậy ta thấy : 

                                $\int_{0}^{\infty }e^{-\frac{t^{2}}{2}}cos\frac{x^{2}}{2t^{2}}dt=\sqrt{\frac{\pi }{2}}e^{-\frac{x}{\sqrt{2}}}cos\frac{x}{\sqrt{2}}$

    Ví dụ trên tương đối đơn giản và là một hình dung cụ thể cho thủ thuật tính tích phân thông qua phương trình vi phân




#555864 Chứng minh tích phân bằng tổ hợp

Posted by sinh vien on 23-04-2015 - 19:37 in Giải tích

Bài toán.(Putnam ? ) Hỏi với giá trị của n như thế nào thì tích phân

    $I=\int_{0}^{2\pi }cosxcos2x...cosnxdx$ có giá trị khác không ? , 

Lời giải .

Áp dụng định lí Morvie : $e^{ix}=cosx+isinx$ ta dễ dàng suy ra $cosx=\frac{1}{2}(e^{ix}+e^{-ix})$ thay vào biểu thức của $I$ ta thấy

$I=\frac{1}{2^{n}}\int_{0}^{2\pi }(e^{ix}+e^{-ix})(e^{2ix}+e^{-2ix})...(e^{nix}+e^{-nix})dx=\frac{1}{2^{n}}\int_{0}^{2\pi }\sum_{\varepsilon _{1},\varepsilon _{2}...\varepsilon _{n}=\pm 1}e^{(\varepsilon _{1}+2\varepsilon _{2}+...n\varepsilon _{n})ix}dx$=

  $\frac{1}{2^{n}}\sum_{\varepsilon _{1},...\varepsilon _{n}=\pm 1}\int_{0}^{2\pi }e^{(\epsilon _{1}+2\varepsilon _{2}+...+n\varepsilon _{n})ix}dx$

  Ta có :   $\int_{0}^{2\pi }e^{mix}dx=2\pi$ nếu $m=0$ và bằng 0 trong trường hợp $m\neq 0$. 

Do đó: $I=\frac{\pi}{2^{n-1}}S(n)$

trong đó $S(n)$ là số cách chọn các dấu +, -  sao cho $\pm 1\pm 2\pm...\pm n=0$.(Do n=0 hiển nhiên làm cho tích phân ban đầu khác không nên ở đây ta chỉ xét các giá trị $n\geq 1$ )

    Bài toán của ta trở thành tìm n sao cho $S(n)\neq 0$.    

Ta xét các trường hợp của n

 Nếu $n=4k$ thì từ đẳng thức : $(1-2-3+4)+(5-6-7+8)+...+((4k-3)-(4k-2)-(4k-1)+4k)=0$ ta suy ra $S(n)\neq 0$  

Nếu n=4k+3  thì từ đẳng thức: $(1+2-3)+(4-5-6+7)+...+((4k)-(4k+1)-(4k+2)+(4k+3))=0$ ta cũng suy ra $S(n)\neq 0$

Nếu n=4k+1 hoặc 4k+2:

   $\varepsilon _{1}+2\varepsilon _{2}+...+n\varepsilon _{n}\equiv 1+2+...+n=\frac{n(n+1)}{2}\not\equiv 0 mod2$  , trong đó $\varepsilon _{1},...\varepsilon _{n}=\pm 1$ nên $S(n)=0$

  Từ khảo sát trên ta dễ thấy giá trị cần tìm là n=4k , n=4k+3 , trong đó k là số tự nhiên nào đó

 

  




#555995 Khai triển Fourier bằng lượng giác và số phức

Posted by sinh vien on 24-04-2015 - 10:23 in Giải tích

Bài toán. Tìm khai triển Fourier của hàm $f(x)=(\frac{sinnx}{sinx})^{2}$

Lời giải. Từ công thức Euler: $e^{ix}=cosx+isinx$ và đằng thức sau:

    $\sum_{k=1}^{n}e^{2ikx}=\frac{e^{2i(n+1)x}-1}{e^{2ix}-1}$  ( chứng minh khá đơn giản nên mình không đưa ra ở đây )

Tách lấy phần thực và phần ảo ta thu được hai hệ thức quan trọng sau :

$cos2x+cos4x+...+cos2nx=\frac{sinnxcos(n+1)x}{sinx}$

$sin2x+sin4x+...+sin2nx=\frac{sinnxsin(n+1)x}{sinx}$

 Ta viết lại hàm $f(x)$ lai dưới dạng 

 $(\frac{sinnx}{sinx})^{2}=\left ( \frac{sinnxsin(n+1)x}{sinx} \right )^{2}+\left ( \frac{sinnxcos(n+1)x}{sinx} \right )^{2}$.

 Thay kết quả thu được vào và chú ý đến hằng đẳng thức Lagrange: 

$\left ( x_{1}+x_{2}+...x_{n} \right )^{2}=\sum_{i=1}^{n}x^{2}_{i}+2\sum_{1\leqslant k< l\leqslant n}x_{k}x_{l}$, ta thu được  :

$f(x)=n+2\sum_{1\leq k< l\leq n}(sin2kxsin2lx+cos2kxcos2lx)$

 

$=n+2\sum_{1\leq l< k\leq n}cos2(k-l)x$

$=n+2\sum_{m=1}^{n-1}(n-m)cos2mx$.

 Do đó ta có khai triển Fourier

    $\left ( \frac{sinnx}{sinx} \right )^{2}=n+2\sum_{m=1}^{n-1}(n-m)cos2mx$.

 Lưu ý ở các bước cuối bạn cần một chút kiến thức tổ hợp nhỏ.




#557802 Sự kết hợp giữa tích phân và số phức

Posted by sinh vien on 04-05-2015 - 08:08 in Giải tích

Bài toán. Chứng minh rằng số điểm nguyên trong hình lập phương đóng $-n\leq x,y,z\leq n$ thỏa mãn điều kiện $-s\leq x+y+z\leq s$ bằng $\frac{1}{2\pi }\int_{-\pi }^{\pi }\left ( \frac{sin\frac{2n+1}{2}t}{sin\frac{t}{2}} \right )^{3}\frac{sin\frac{2s+1}{2}t}{sin\frac{t}{2}}dt$ trong đó s, n nguyên

Lời giải. Xét hàm sinh của bài toán

     $G(x)=\left ( \frac{1}{x^{n}}+\frac{1}{x^{n-1}}+...+\frac{1}{x}+1+x+...+x^{n-1}+x^{n} \right )^{3}$

  $=...+a_{-k}x^{-k}+a_{-k+1}x^{-k+1}+...+a_{-1}x^{-1}+a_{0}+a_{1}x+...+a_{k-1}x^{k-1}+a_{k}x^{k}+...$

 Dễ thấy khi đó số các điểm nguyên của hình lập phương đóng thỏa mãn điều kiện $x+y+z=m$ trong đó m nguyên và $-3n\leq m\leq 3n$ chính là hệ số $a_{m}$ trong khai triển trên.

 Do đó 

  Số cần tìm = $a_{-s}+a_{-s+1}+...a_{1}+a_{0}+a_{1}+...+a_{s-1}+a_{s}$

 Để tính tổng này ta chú ý đến một kết quả quen thuộc trong giải tích phức:

                         $\int_{-\pi }^{\pi }e^{ikx}dx=2\pi$ nếu k=0 và bằng 0 trong các trường hợp còn lại.

nên nói chung ta thấy:

$\frac{1}{2\pi }\int_{-\pi }^{\pi }G(e^{it})e^{-kxt}dt=a_{k}$, trong đó$-3n\leq k\leq 3n$

 Để thuận tiện trong trình bày ta sẽ đặt $\zeta =e^{it}$ nên ta được :

         $\frac{1}{2\pi }\int_{-\pi }^{\pi }G(\zeta)\zeta ^{-k}dt=a_{k}$

 Ta có

  $\sum_{i=-s}^{s}a_{i}=\frac{1}{2\pi }\int_{-\pi }^{\pi }G(\zeta)(\sum_{i=-s}^{s}\zeta ^{i})dt$

  Không quá khó khăn để chứng minh đẳng thức sau:

                 $\sum_{i=-m}^{m}\zeta ^{i}=\frac{\zeta ^{-\frac{2m+1}{2}}-\zeta ^{-\frac{2m+1}{2}}}{\zeta ^{-\frac{1}{2}}-\zeta ^{\frac{1}{2}}}=\frac{sin\frac{2m+1}{2}t}{sin\frac{t}{2}}$   ( chú ý ta có: $\zeta =e^{it}$

nên $\sum_{i=-s}^{s}a_{i}=\frac{1}{2\pi }\int_{-\pi }^{\pi }\left ( \frac{sin\frac{2n+1}{2}t}{sin\frac{t}{2}} \right )^{3}\frac{sin\frac{2s+1}{2}t}{sin\frac{t}{2}}dt$ (đpcm)




#557803 Tính tích phân: $\iiint {x^2+y^2+z^2} dxdydz $, V là...

Posted by sinh vien on 04-05-2015 - 09:55 in Giải tích

Đặt $x=\frac{1}{2}+rsin\varphi cos\theta ;y=\frac{1}{2}+rsin\varphi sin\theta ;z=\frac{1}{2}+rcos\varphi$;

trong đó $0\leq r\leq \frac{\sqrt{3}}{2};0\leq \theta \leq 2\pi ;0\leq \varphi \leq \pi$. Định thức hàm $J=r^{2}sin\varphi$

 Do đó $\int_{0}^{2\pi }d\theta \int_{0}^{\pi }d\varphi \int_{0}^{\frac{\sqrt{3}}{2}}(\frac{3}{2}+rsin\varphi cos\theta +rsin\varphi sin\theta +rcos\varphi )r^{2}sin\varphi dr$. Tách tích phân trên thành 4 phần:

$I_{1}=\frac{3}{2}\int_{0}^{2\pi }d\theta \int_{0}^{\pi }sin\varphi d\varphi \int_{0}^{\frac{\sqrt{3}}{2}}r^{2}dr$ 

$I_{2}=\int_{0}^{2\pi }cos\theta d\theta \int_{0}^{\pi }sin^{2}\varphi d\varphi \int_{0}^{\frac{\sqrt{3}}{2}}r^{3}dr$

$I_{3}=\int_{0}^{2\pi }sin\theta d\theta \int_{0}^{\pi }sin^{2}\varphi d\varphi \int_{0}^{\frac{\sqrt{3}}{2}}r^{3}dr$

$I_{4}=\int_{0}^{2\pi }d\theta \int_{0}^{\pi }cos\varphi sin\varphi d\varphi \int_{0}^{\frac{\sqrt{3}}{2}}r^{3}dr$.

 Ta thấy $\int_{0}^{2\pi }cos\theta d\theta =\int_{0}^{2\pi }sin\theta d\theta =\int_{0}^{\pi }cos\varphi sin\varphi d\varphi =0$ nên

 

$I=\frac{3}{2}\times 2\pi \times 2\times \frac{\sqrt{3}}{8}$




#557829 Tính tích phân hai lớp bằng tổng Rieman $I=\int \int_{D...

Posted by sinh vien on 04-05-2015 - 17:59 in Giải tích

Bài toá n. Tính tích phân

   

    $I=\int \int_{D} ln\left | sin(x-y) \right |dxdy$, trong đó $D=\left \{ (x,y)/0\leq x< y\leqslant \pi \right \}$

Lời giải:

    Để tính tích phân hai lớp này, trước hết ta chứng minh đẳng thức sau:

   $\sum_{0\leqslant j< k\leq n-1}\frac{\pi ^{2}}{n^{2}}ln\left | sin\left ( \frac{j\pi }{n}-\frac{k\pi }{n} \right ) \right |=\frac{\pi ^{2}}{2n}lnn-\left ( 1-\frac{1}{n} \right )\frac{\pi ^{2}}{2}ln2$.

  Xét định thức:

     $A=\begin{vmatrix} 1 & 1 & 1 & ... & 1\\ 1& \varepsilon &\varepsilon ^{2} &...& \varepsilon ^{n-1}\\ 1& \varepsilon ^{2} &\varepsilon ^{4} &... &\varepsilon ^{2(n-1)} \\ ... & ... & .... & ... & ...\\ 1 & \varepsilon ^{n-1} &\varepsilon ^{2(n-1)} &... &\varepsilon ^{(n-1)(n-1)} \end{vmatrix}$

   trong đó $\varepsilon =e^{\frac{2\pi i}{n}}$

 Ta thấy đây là định thức Vandermonde nên 

  $A=\prod_{0\leq j< k\leq n-1}\left ( \varepsilon ^{j}-\varepsilon ^{k} \right )$.

 Mặt khác  liên hợp :$\bar{A}=\begin{vmatrix} 1 & 1 & 1 & ... & 1\\ 1& \bar{\varepsilon } &\bar{\varepsilon }^{2} &... &\bar{\varepsilon}^{n-1} \\1 & \bar{\varepsilon }^{2} & \bar{\varepsilon } ^{4}& ... & \bar{\varepsilon }^{2(n-1)}\\ ... & ... & ...& ... & \\ 1& \bar{\varepsilon }^{n-1} &\bar{\varepsilon }^{2(n-1)} &... & \bar{\varepsilon }^{(n-1)(n-1)} \end{vmatrix}$

 Nên

          $A\bar{A}=\begin{vmatrix} n& 0 & 0 &... &0 \\ 0 & n & 0 & ...& 0\\... & ...& ...& ...& ...\\ ... & ... & ... & ... &... \\ 0& 0& 0 & ... &n \end{vmatrix}=n^{n}$

$\Rightarrow \prod_{0\leq j< k\leq n-1}\left ( \varepsilon ^{j}-\varepsilon ^{k} \right )\prod_{0\leq j< k\leq n-1}\overline{(\varepsilon ^{j}-\varepsilon ^{k})}=n^{n}$

$\prod_{0\leq j< k\leq n-1}\left | \varepsilon ^{j}-\varepsilon ^{k}\right |^{2}=n^{n}$.

 Ta thấy:

     $\left |\varepsilon ^{j} -\varepsilon ^{k} \right |^{2}=\left ( cos\frac{2\pi j}{n} -cos\frac{2\pi k}{n}\right)^{2}+\left (sin\frac{2\pi j}{n} -sin\frac{2\pi k}{n}\right )^{2}$

        $=2\left ( 1-cos\frac{2\pi (j-k)}{n} \right )=\left [ 2sin\left ( \frac{j\pi }{n}-\frac{k\pi }{n} \right ) \right ]^{2}$

    hay $\prod_{0\leq j< k\leq n-1}\left [ 2sin\left ( \frac{j\pi }{n}-\frac{k\pi }{n} \right )\right ]^{2}=n^{n}$

  Lấy logarithm hai vế và sử dụng một chút biến đổi ta được đẳng thức đã nêu.

   Dễ dàng nhận thấy rằng vế trái của đẳng thức này là tổng Riemann của tích phân cần tính ở đây ta đã sử dụng  phân hoạch đều:

     Do đó:

  $\int \int_{D}ln\left | sin(x-y) \right |dxdy=lim_{n\rightarrow \infty }\left [ \frac{\pi ^{2}}{2n} lnn-\left ( 1-\frac{1}{n} \right )\frac{{\pi ^{2}}}{2}ln2\right ]$$=-\frac{\pi ^{2}}{2}ln2$.

  




#557831 Một sự kết hợp giữa đại số tuyến tính và tổ hợp

Posted by sinh vien on 04-05-2015 - 18:29 in Đại số tuyến tính, Hình học giải tích

Bài toán.  Cho $k\leq \frac{n}{2}$ và $F$ là một họ các ma trận con của một ma trận $n\times n$  sao cho hai ma trận con bất kì đều giao nhau ( có những phần tử chung ) thì $\left | F \right |\leq \left ( C_{n-1}^{k-1} \right )^{2}$, trong đó $\left | A \right |$ kí hiệu số phần tử của tập A.

Lời giải.

Với mọi ma trận con M thuộc họ F, đặt $R_{M};C_{M}$ là các bộ k- số  chỉ thứ tự của các hàng và các cột . Dể dàng nhận thấy $R_{M};C_{M}$ xác định duy nhất ma trận M. Theo giả thiết của bài toán

      $R_{M_{1}}\bigcap R_{M_{2}}\neq \varnothing ;C_{M_{1}}\bigcap C_{M_{2}}\neq \varnothing$ trong đó $M_{1};M_{2}$ là hai phần tử bất kì thuộc họ F.

 Khi đó nếu đặt   $R=\left \{ R_{M};M\in F \right \}$;$C=\left \{ C_{M};M\in F \right \}$ là hai  họ con chứa các bộ k -số lấy từ n số sao cho hai bộ k- số bất kỳ đều có phần tử chung .

   Theo định lý Erdos-Ko-Rado ( xem tuyển tập các chuyên đề tổ hợp ) ta thấy

                 $\left | R \right |\leq C_{n-1}^{k-1};\left | C \right |\leq C_{n-1}^{k-1}\textrm{}$

Theo nhận xét trên ta suy ra$\left | F \right |\leq \left ( C_{n-1}^{k-1}\right )^{2}$ (đpcm )




#557834 CMR tồn tại hằng số B dương thỏa $\sum_{i,j=1}^{n...

Posted by sinh vien on 04-05-2015 - 18:56 in Giải tích

Bài toán. (PUTNAM 2011) Cho $a_{1},a_{2},...a_{n}$ là các số thực thỏa mãn 

          $\int_{-\infty }^{\infty }\left ( \sum_{i=1}^{n}\frac{1}{1+\left ( x-a_{i} \right )^{2}}\right )^{2}dx\leqslant An$ với mọi n , trong đó A là một hằng số dương .

 Chứng minh rằng tồn tại hằng số B dương thỏa $\sum_{i,j=1}^{n}\left ( 1+\left ( a_{i}-a_{j} \right )^{2} \right )\geq Bn^{3}$.

Lời giải. 

 Với $a\neq 0$, $\int_{-\infty }^{\infty }\frac{du}{\left ( 1+\left ( u+a \right )^{2} \right )\left ( 1+\left ( u-a \right )^{2} \right )}=\frac{1}{4a(1+a^{2})}\int_{-\infty }^{\infty }\left [ \frac{u+a}{1+\left ( u+a\right )^{2}}-\frac{u-a}{1+\left ( u-a \right )^{2}} \right ]du$$=\frac{1}{4a\left ( 1+a^{2} \right )}\left [ \frac{1}{2}ln\frac{1+\left ( u+a \right )^{2}}{1+(u-a)^{2}} +aarctan(u+a)+aarctan(u-a)\right ]_{-\infty }^{\infty }$$=\frac{\pi }{2(1+a^{2})}$.

   Sử dụng phép đổi biến: $x=u+\frac{a+b}{2}$ ta được 

$\int_{-\infty }^{\infty }\frac{dx}{(1+(x-a)^{2})(1+(x-b)^{2})}=\frac{2\pi }{4+(a-b)^{2}}$, trong đó $a\neq b$

Trong trường hợp a=b 

  $\int_{-\infty }^{\infty }\frac{dx}{(1+(x-a)^{2})^{2}}=\int_{-\frac{\pi }{2}}^{\frac{\pi }{2}}\frac{\frac{1}{cos^{2}\theta }d\theta }{\frac{1}{cos^{4}\theta }}=\int_{-\frac{\pi }{2}}^{\frac{\pi }{2}}cos^{2}\theta d\theta =\frac{\pi }{2}$

 nên côn thức trên cũng đúng với mọi cặp số a,b.

 Ta có

$An\geq \int_{-\infty }^{\infty }\left [ \sum_{i=1}^{n}\frac{1}{1+(x-a_{i})^{2}}\right ]^{2}dx=\sum_{i,j=1}^{n}\int_{-\infty }^{\infty }\frac{dx}{(1+(x-a_{i})^{2})(1+(x-a_{j})^{2})}$$=\sum_{i,j=1}^{n}\frac{2\pi }{4+(a_{i}-a_{j})^{2}}\geq \frac{\pi }{2}\sum_{i,j=1}^{n}\frac{1}{1+\left ( a_{i}-a_{j} \right )^{2}}$

 Sử dụng bất đẳng thức quen thuộc Cauchy- Schwarz 

$n^{4}\leq \left [ \sum_{i,j=1}^{n}\frac{1}{1+(a_{i}-a_{j})^{2}} \right ]\sum_{i,j=1}^{n}(1+(a_{i}-a_{j})^{2})\leq \frac{2An}{\pi }\sum_{i,j=1}^{n}(1+(a_{i}-a_{j})^{2})$

  Chọn $B=\frac{\pi }{2A}$ . thì điều kiện bài toán được thỏa mãn




#557905 Ứng dụng giải tích trong các bài toán số học

Posted by sinh vien on 05-05-2015 - 08:46 in Giải tích

Bài toán.  Cho n là một số nguyên lớn hơn bằng 2.

Chứng minh rằng

 $\sum_{p\leq n}\frac{1}{p}> lnlnn-1$, trong đó ta quy ước p chỉ các số nguyên tố nên tổng ở đây lấy trên tập các số nguyên tố nhỏ hơn bằng n.

Lời giải. Gọi A(n) là tập các số nguyên dương sao cho các ước số nguyên tố nhỏ hơn hoặc bằng n.

Khi đó

 $\prod_{p\leq n}\left ( 1+\frac{1}{p}+\frac{1}{p^{2}}+...+\frac{1}{p^{k}}+.... \right )=\sum_{m\in A(n)}\frac{1}{m}$ (1)

Ta thấy rằng $\sum_{m\in A(n)}\frac{1}{m}\geq \sum_{m=1}^{n}\frac{1}{m}$ (định nghĩa của tập A(n) )

mà ta lại có bất đẳng thức sau ( chứng minh không quá khó khăn )   $\sum_{m=1}^{n}\frac{1}{m}> lnn$  . Lưu ý rằng ta cũng có đẳng thức sau :$1+\frac{1}{p}+\frac{1}{p^{2}}+...+\frac{1}{p^{k}}+...=\left (1-\frac{1}{p} \right )^{-1}$ nên ta có thể viết lại  (1) như sau$\prod_{p\leq n}\left ( 1-\frac{1}{p} \right )^{-1}> lnn$.

 Để thực hiện tiếp phép chứng minh ta cần đến một bất đẳng thức không tầm thường :

$e^{t+t^{2}}\geq \left ( 1-t \right )^{-1}, 0\leq t\leq \frac{1}{2}$ có thể được chứng minh dễ dàng bằng  phương pháp lấy đạo hàm :

  Do đó :

    $\prod_{p\leq n}e^{\frac{1}{p}+\frac{1}{p^{2}}}\geq \prod_{p\leq n}\left ( 1-\frac{1}{p} \right )^{-1}> lnn$.(2)

 Từ (2) không quá khó để nhận ra rằng $\sum_{p\leq n}\frac{1}{p}+\sum_{p\leq n}\frac{1}{p^{2}}>lnlnn\Rightarrow \sum_{p\leq n}\frac{1}{p}> lnlnn-\sum_{p\leq n}\frac{1}{p^{2}}$

 mà $\sum_{p\leq n}\frac{1}{p^{2}}\leq \sum_{k=2}^{\infty }\frac{1}{k^{2}}=\frac{\pi ^{2}}{6}-1< 1$ (3)

  Từ (2) và (3) ta đi đến :$\sum_{p\leq n}\frac{1}{p}> lnlnn-1$ (đpcm)

         




#557941 Khai triển Fourier cho hàm Logaritm Gamma - Công thức Kummer

Posted by sinh vien on 05-05-2015 - 15:39 in Giải tích

Bài toán.Chứng minh rằng :

   $ln\Gamma (x)=\frac{1}{2}ln(2\pi )+\frac{1}{2}\sum_{k=1}^{\infty }\frac{cos2k\pi x}{k}+\sum_{k=1}^{\infty }\frac{(\gamma +ln2\pi k)sin2k\pi x }{\pi k}, (0< x< 1)$, trong đó $\gamma$ là hằng số Euler

Lời giải.  Để thực hiện được phép chứng minh trên ta cần đến các kết quả quen thuộc sau :

 $\Gamma (1-x)\Gamma (x)=\frac{\pi }{sin(\pi x)}$ ; $\int_{0}^{\pi }ln(sinx)dx=-\pi ln2$ và công thức

$\Gamma (x)=\frac{e^{-\gamma x}}{x}\prod_{k=1}^{\infty }e^{\frac{x}{k}}\left ( 1+\frac{x}{k} \right )^{-1}$

  Ta có:

 $a_{0}=2\int_{0}^{1}ln\Gamma (x)dx=2\int_{1}^{0}ln\Gamma (1-x)d(1-x)=2\int_{0}^{1}ln\Gamma (1-x)dx$

$\Rightarrow a_{0}=\int_{0}^{1}ln\left ( \frac{\pi }{sin\pi x} \right )dx=ln\pi -\int_{0}^{1}ln(sin\pi x)dx=ln(2\pi )$

$a_{k}=2\int_{0}^{1}ln\Gamma (x)cos2k\pi xdx=2\int_{0}^{1}ln\Gamma (1-x)cos2k\pi xdx$

$\Rightarrow a_{k}=\int_{0}^{1}ln\left ( \frac{\pi }{sin\pi x} \right )cos2k\pi xdx=-\int_{0}^{1}ln(sin\pi x)cos2k\pi xdx$$=-\frac{1}{\pi }\int_{0}^{\pi }ln(sinx)cos2kxdx=-ln(sinx)\frac{sin2k x}{2k\pi } _{0}^{\pi } +\frac{1}{2k\pi }\int_{0}^{\pi }\frac{sin2k xcosx}{sinx}dx$=$\frac{1}{2k\pi }\int_{0}^{\pi }\frac{sin2kxcosxdx}{sinx}$

Đặt $I_{k}=\int_{0}^{\pi }\frac{sin2kxcosxdx}{sinx}\Rightarrow I_{k+1}=\int_{0}^{\pi }\frac{sin2(k+1)xcosxdx}{sinx}$. Do đó $I_{k+1}-I_{k}=2\int_{0}^{\pi }\frac{cos(2k+1)xsinxcosxdx}{sinx}=2\int_{0}^{\pi }cos(2k+1)xcosxdx=0$ nên $I_{k}=2\int_{0}^{\pi }cos^{2}xdx=\pi \Rightarrow a_{k}=\frac{1}{2k\pi }\times \pi =\frac{1}{2k}$

  Theo một kết quả quen thuộc:

                              $\Gamma (x)=\frac{e^{-\gamma x}}{x}\prod_{k=1}^{\infty }e^{\frac{x}{k}}\left ( 1+\frac{x}{k} \right )^{-1}\Rightarrow ln\Gamma (x)=-(\gamma x+lnx)+\sum_{k=1}^{\infty }\left \{ \frac{x}{k}-ln\left ( 1+\frac{x}{k} \right ) \right \}$$\Rightarrow ln\Gamma (x)sin2k\pi x=-(\gamma x+lnx)sin2k\pi x+\sum_{k=1}^{\infty }\left \{ \frac{x}{k}-ln\left ( 1+\frac{x}{k} \right ) \right \}sin2k\pi x$.

  trong đó :

       $\int_{0}^{1}(-\gamma x-lnx)sin2k\pi xdx=\frac{\gamma xcos2k\pi x}{2k\pi }_{0}^{1}-\frac{\gamma }{2k\pi }\int_{0}^{1}cos2k\pi xdx+\frac{lnxcos2k\pi x}{2k\pi }_{0}^{1}-\frac{1}{2k\pi }\int_{0}^{1}\frac{cos2k\pi xdx}{x}=\frac{\gamma }{2k\pi }+\left [ \frac{lnxcos2k\pi x-Ci(2k\pi x)}{2k\pi } \right ]_{0}^{1}$$\frac{\gamma -Ci(2k\pi )}{2k\pi }-lim_{\epsilon \rightarrow \infty }\frac{ln\epsilon -Ci(2k\pi \epsilon )}{2k\pi }=\frac{\gamma -Ci(2k\pi )}{2k\pi }-lim_{\epsilon \rightarrow \infty }\frac{ln\epsilon -ln(2k\pi )-ln\epsilon -\gamma +O(1)}{2k\pi }$$=\frac{2\gamma +ln(2k\pi )-Ci(2k\pi )}{2k\pi }$

      $\int_{0}^{1}\sum_{k=1}^{\infty }\left \{ \frac{x}{k}-ln(1+\frac{x}{k}) \right \}sin2k\pi xdx=-\frac{1}{2k\pi }\sum_{n=1}^{\infty }\left \{ \frac{1}{n}+Ci(2k\pi n+2k\pi )-Ci(2k\pi n)-ln\left ( 1+\frac{1}{n} \right ) \right \}$

$=-\frac{1}{2k\pi }\left [ \sum_{n=1}^{\infty } \left \{ \frac{1}{n}-ln\left ( 1+\frac{1}{n} \right ) \right \}+lim_{N\rightarrow \infty }\left ( \sum_{n=2}^{N+1}Ci(2k\pi n)-\sum_{n=1}^{N}Ci(2k\pi n) \right )\right ]$

$=-\frac{1}{2k\pi }\left [ \gamma +lim_{N\rightarrow \infty } \left ( Ci(2k\pi x+2k\pi N) -Ci(2k\pi )\right )\right ]=\frac{Ci(2k\pi )-\gamma }{2k\pi }$.

  Do đó

                $\int_{0}^{1}ln\Gamma (x)sin2k\pi xdx=\frac{\gamma +ln(2k\pi )}{2k\pi }\Rightarrow b_{k}=\frac{\gamma +ln(2k\pi )}{k\pi }$




#558170 Hệ phương trình vi phân loại khó

Posted by sinh vien on 07-05-2015 - 11:55 in Giải tích

Bài toán ( Putnam 2009) . Gỉa sử $f,g,h$ là các hàm khả vi trên một khoảng mở chứa điểm 0 Và thỏa mãn :

$f^{'}=2f^{2}gh+\frac{1}{gh};g^{'}=fg^{2}h+\frac{4}{fh};h^{'}=3fgh^{2}+\frac{1}{fg}$ 

 và $f(0)=1;g(0)=1;h(0)=1$.

 Tìm một biểu thức cho hàm f(x) , giả thiết f(x) trong triệt tiêu trên khoảng mở chứa 0.

Lời giải. Nhân đẳng thức thứ nhất cho gh , thứ 2 cho fh, và đẳng thức cuối cho fg ta được

    $f^{'}gh=2\left ( fgh \right )^{2}+1;fg^{'}h=(fgh)^{2}+4;fgh^{'}=3(fgh)^{2}+1$.

 Cộng các đẳng thức này ta được

    $f^{'}gh+fg^{'}h+fgh^{'}=6(fgh)^{2}+6\Rightarrow (fgh)^{'}=6(fgh)^{2}+6$.

 Bằng cách đặt k(x)=f(x)g(x)h(x); ta được $k^{'}=6k^{2}+6\Rightarrow \frac{\mathrm{d} k}{\mathrm{d} x}=6k^{2}+6\Rightarrow \frac{dk}{6k^{2}+6}=dx$

 Kết hợp với điều kiện k(0)=1 ta thấy: $k(x)=tan\left ( 6x+\frac{\pi }{4} \right ).$

   Lại thấy:$\frac{f^{'}}{f}=2tan\left ( 6x+\frac{\pi }{4} \right )+cot\left ( 6x+\frac{\pi }{4} \right )$

$\Rightarrow lnf(x)=\frac{-2lncos\left ( 6x+\frac{\pi }{4} \right )+lnsin\left ( 6x+\frac{\pi }{4} \right )}{6}$

$\Rightarrow f(x)=e^{c}\left ( \frac{sin\left ( 6x+\frac{\pi }{4} \right )}{cos^{2}\left ( 6x+\frac{\pi }{4} \right )} \right )^{\frac{1}{6}}$. Từ giả thiết f(0)=1 ta tính được

   $f(x)=\frac{1}{2^{12}}\left ( \frac{sin\left ( 6x+\frac{\pi }{4} \right )}{cos^{2}\left ( 6x+\frac{\pi }{4} \right )} \right )^{\frac{1}{6}}$.

  Lưu ý là ta cũng có thể tính được g(x), h(x) bằng các tương tự.




#558174 tuyển chọn các bài toán tính định thức

Posted by sinh vien on 07-05-2015 - 12:35 in Đại số tuyến tính, Hình học giải tích

Bài toán:(Putnam 2009 )Cho n > 3, n nguên dương. Tính định thức:

 $D_{n}=\begin{vmatrix} cos1 &cos2 &... &cosn \\ cos(n+1) & cos(n+2) &... &cos2n \\... &... &... &... \\cos(n^{2}-n+1) & cos(n^{2}-n+2) &... &cosn^{2} \end{vmatrix}$

Lời giải.  Lấy cột thứ 3 + cột thứ 1$\rightarrow$ cột thứ 1 và sử dụng biến đổi lượng giác , ta được :

$D_{n}=\begin{vmatrix} 2cos2cos1 &cos2 &... & ... &... cos(n) \\2cos(n+2)cos1 &cos(n+2) &... & ... &cos2n \\... & ... & & ... &... \\... &... & ... & ... &... \\2cos(n^{2}-n+2)cos1 & cos(n^{2}-n+2) &... &... & cosn^{2} \end{vmatrix}$

   Lúc này cột thứ 1= 2cos1 cột thứ 2 $\Rightarrow$ det$D_{n}=0$.

Bài toán . Xét các số phức$z_{1},z_{2},...,z_{2n}$ thỏa mãn điều kiện  $\left | z_{1} \right |=\left | z_{2} \right |=...=\left | z_{n+3} \right |$ và $argz_{1}\geqslant argz_{2}\geqslant ...\geq argz_{n+3}$.

  Tính định thức của ma trận B ,trong đó  $b_{ij}=\left | z_{i}-z_{j+n} \right |,i,j\in \left \{ 1,2,...n \right \}$.

 Lời giải . Ta có nhận xét sau bài toán đề cập đến modun và argument của một số phức nên ta sẽ sử dụng dạng lượng giác của số phức trong các tính toán. 

    Với hai số phức $z=r(cosx+isinx);\omega =r(cosy+isiny)\Rightarrow \left | z-\omega \right |=2r\left | sin\left ( \frac{x-y}{2} \right ) \right |$. 

  Kết hợp với điều kiện ở đầu bài, ta thấy B có dạng như sau:

$B=(2r)^{n}\begin{vmatrix} sin(x_{1}-x_{n+1}) &sin(x_{1}-x_{n+2}) & sin(x_{1}-x_{n+3}) & ...\\... &... & ... &... \\ sin(x_{n}-x_{n+1}) & sin\left ( x_{n}-x_{n+2} \right ) & sin(x_{n}-x_{n+3}) & ... \end{vmatrix}$

 trong đó $x_{i}$ kí hiệu tương ứng argument của $z_{i}$ và r=$\left | z_{1} \right |=\left | z_{2} \right |=...=\left | z_{2n} \right |$

 Nếu ta tách det B theo các cột thì dễ dàng nhận thấy det B=0 

 

 




#558175 tuyển chọn các bài toán tính định thức

Posted by sinh vien on 07-05-2015 - 12:35 in Đại số tuyến tính, Hình học giải tích

Bài toán:(Putnam 2009 )Cho n > 3, n nguên dương. Tính định thức:

 $D_{n}=\begin{vmatrix} cos1 &cos2 &... &cosn \\ cos(n+1) & cos(n+2) &... &cos2n \\... &... &... &... \\cos(n^{2}-n+1) & cos(n^{2}-n+2) &... &cosn^{2} \end{vmatrix}$

Lời giải.  Lấy cột thứ 3 + cột thứ 1$\rightarrow$ cột thứ 1 và sử dụng biến đổi lượng giác , ta được :

$D_{n}=\begin{vmatrix} 2cos2cos1 &cos2 &... & ... &... cos(n) \\2cos(n+2)cos1 &cos(n+2) &... & ... &cos2n \\... & ... & & ... &... \\... &... & ... & ... &... \\2cos(n^{2}-n+2)cos1 & cos(n^{2}-n+2) &... &... & cosn^{2} \end{vmatrix}$

   Lúc này cột thứ 1= 2cos1 cột thứ 2 $\Rightarrow$ det$D_{n}=0$.

Bài toán . Xét các số phức$z_{1},z_{2},...,z_{2n}$ thỏa mãn điều kiện  $\left | z_{1} \right |=\left | z_{2} \right |=...=\left | z_{n+3} \right |$ và $argz_{1}\geqslant argz_{2}\geqslant ...\geq argz_{n+3}$.

  Tính định thức của ma trận B ,trong đó  $b_{ij}=\left | z_{i}-z_{j+n} \right |,i,j\in \left \{ 1,2,...n \right \}$.

 Lời giải . Ta có nhận xét sau bài toán đề cập đến modun và argument của một số phức nên ta sẽ sử dụng dạng lượng giác của số phức trong các tính toán. 

    Với hai số phức $z=r(cosx+isinx);\omega =r(cosy+isiny)\Rightarrow \left | z-\omega \right |=2r\left | sin\left ( \frac{x-y}{2} \right ) \right |$. 

  Kết hợp với điều kiện ở đầu bài, ta thấy B có dạng như sau:

$B=(2r)^{n}\begin{vmatrix} sin(x_{1}-x_{n+1}) &sin(x_{1}-x_{n+2}) & sin(x_{1}-x_{n+3}) & ...\\... &... & ... &... \\ sin(x_{n}-x_{n+1}) & sin\left ( x_{n}-x_{n+2} \right ) & sin(x_{n}-x_{n+3}) & ... \end{vmatrix}$

 trong đó $x_{i}$ kí hiệu tương ứng argument của $z_{i}$ và r=$\left | z_{1} \right |=\left | z_{2} \right |=...=\left | z_{2n} \right |$

 Nếu ta tách det B theo các cột thì dễ dàng nhận thấy det B=0 

 

 




#558201 ứng dụng số phức để giải hệ phương trình vi phân

Posted by sinh vien on 07-05-2015 - 16:43 in Giải tích

Bài toán.  Giải hệ phương trình vi phân 

                                    $x^{''}-y^{'}+x=0$

                                    $y^{''}+x^{'}+y=0$

trong đó x(t) , y(t) là các hàm nhận giá trị  thực.

Lời giải. Nhân phương trình thứ hai cho i và cộng với phương trình thứ nhất ta được

$\left ( x+iy \right )^{''}+i\left ( x+iy \right )^{'}+\left ( x+iy \right )=0$

Bằng cách đặt  $z=x+iy$ đẳng thức trên trở thành một phương trình vi phân cấp 2 theo biến hàm mới - z.

                                                   $z^{''}+iz^{'}+z=0$

 Phương trình đặc trưng $\lambda ^{2}+i\lambda +1=0\Rightarrow \lambda _{1,2}=\frac{-1\pm \sqrt{5}}{2}i$

  Nghiệm tổng quát của phương trình có dạng

 $z(t)=\left ( a+ib \right )exp\left ( \frac{-1+\sqrt{5}}{2}it \right )+\left ( c+id \right )exp\left ( \frac{-1-\sqrt{5}}{2}it \right )$

          Lưu ý rằng ta đang xét x(t), y(t) là các hàm nhận giá trị thực. Thực hiện khai triển và đối chiếu phần thực , phần ảo ta thu được dạng tồng quát của nghiệm phương trình ban đầu

  $x(t)=acos\frac{-1+\sqrt{5}}{2}t-bsin\frac{-1+\sqrt{5}}{2}t+ccos\frac{-1-\sqrt{5}}{2}t-dsin\frac{-1-\sqrt{5}}{2}t$

$y(t)=asin\frac{-1+\sqrt{5}}{2}t+bcos\frac{-1+\sqrt{5}}{2}t+csin\frac{-1-\sqrt{5}}{2}t+dcos\frac{-1-\sqrt{5}}{2}t$

   




#558204 $\int \int \int_{B}\frac{x^{4}+2y^{4}}{x^{4}+4y^{4}+...

Posted by sinh vien on 07-05-2015 - 17:00 in Giải tích

Bài toán. Tính tích phân 

                                               I=  $\int \int \int_{B}\frac{x^{4}+2y^{4}}{x^{4}+4y^{4}+z^{4}}dxdydz$

trong đó $B=\left \{ (x,y,z);x^{2}+y^{2}+z^{2}\leq 1 \right \}$

Bài toán. Tính tích phân

                   I= $\int \int_{D}\left | xy \right |dxdy$

trong đó$D=\left \{ (x,y);x\geq 0,\left ( \frac{x^{2}}{a^{2}}+\frac{y^{2}}{b^{2}} \right )^{2}\leq \frac{x^{2}}{a^{2}}-\frac{y^{2}}{b^{2}} \right \}$

Bài toán. Tính tích phân

          I= $\int_{0}^{1}\int_{0}^{1}\int_{0}^{1}\frac{dxdydz}{\left ( 1+x^{2}+y^{2}+z^{2} \right )^{2}}$

Bài toán.Tính tích phân

     I= $\int_{-\infty }^{\infty }\int_{-\infty }^{\infty }\frac{dxdy}{\left ( p+ax^{2} +2bxy+cy^{2}\right )^{2}}$

trong đó p >0,a>0 và ac-b$^{2}$>0.




#558374 Danh sách các bài toán tích phân - chuỗi - tích vô hạn của Gradshteyn Ryzhik

Posted by sinh vien on 08-05-2015 - 21:16 in Giải tích

Mình post file lên trước và sẽ chứng minh một số kết quả sau.

Attached File  Gradshteyn, Ryzhik. Tables of integrals, series, and products (5ed., AP, 1996)(1762s).pdf   4.61MB   814 downloads




#558415 Danh sách các bài toán tích phân - chuỗi - tích vô hạn của Gradshteyn Ryzhik

Posted by sinh vien on 09-05-2015 - 08:25 in Giải tích

Bài toán . Tính tích phân

I=$\int_{0}^{\infty }e^{-x^{2}}cosaxdx$, trong đó a là hằng số dương

Lời giải . Khai triển Taylor hàm cosax

$cosax=1-\frac{\left ( ax \right )^{2}}{2}+\frac{\left ( ax \right )^{4}}{4!}+...+\frac{(-ax)^{2n}}{(2n)!}+...$

  Đặt $I_{n}=\int_{0}^{\infty }e^{-x^{2}}x^{2n}dx$. 

Ta thấy

$I_{n-1}=\int_{0}^{\infty }e^{-x^{2}}x^{2n-2}dx=\frac{e^{-x^{2}}x^{2n-1}}{2n-1}_{0}^{\infty }+\frac{2}{2n-1}\int_{0}^{\infty }e^{-x^{2}}x^{2n}dx=\frac{2}{2n-1}I_{n-1}$

$\Rightarrow I_{n}=\frac{2n-1}{2}I_{n-1}=\frac{(2n-1)(2n-3)}{4}I_{n-3}=...=\frac{(2n)!}{4^{n}n!}I_{0}$

 mà $I_{0}=\int_{0}^{\infty }e^{-x^{2}}dx=\frac{\sqrt{\pi }}{2}$$\Rightarrow I_{n}=\frac{(2n)!}{4^{n}n!}\frac{\sqrt{\pi }}{2}$

 Do đó 

I=$\frac{\sqrt{\pi }}{2}\sum_{n=0}^{\infty }(-1)^{n}\frac{a^{n}}{(2n)!}\frac{(2n)!}{4^{n}n!}=\frac{\sqrt{\pi }}{2}\sum_{n=0}^{\infty }\frac{\left ( -\frac{a^{2}}{4} \right )^{n}}{n!}=\frac{\sqrt{\pi }}{2}e^{-\frac{a^{2}}{4}}$




#558710 Danh sách các bài toán tích phân - chuỗi - tích vô hạn của Gradshteyn Ryzhik

Posted by sinh vien on 11-05-2015 - 08:03 in Giải tích

Bài toán . Tính tích phân $I=\int_{0}^{\infty }\frac{cosaxdx}{x^{2}+1}$

Lời giải . Ta sẽ sử dụng đến lý thuyết tích phân hàm biến phức. 

Xét tích phân $\int_{\Gamma }\frac{e^{ia\zeta }d\zeta }{\zeta ^{2}+1}$

trong đó $\Gamma =[-R,R]\bigcup\left \{ z,\left | z \right |=R,Imz\geqslant 0,R> 1 \right \}$

Ta viết lại hàm dưới dấu tích phân như sau:

$\frac{e^{ia\zeta }}{\zeta ^{2}+1}=\frac{e^{ia\zeta }}{\zeta +i}\frac{1}{\zeta -i}=\frac{f(\zeta )}{\zeta -i}$

Hàm f(z) chỉnh hình trong và trên $\Gamma$ ,áp dụng định lý Cauchy , ta được

$f(i)=\frac{1}{2\pi i}\int_{\Gamma }\frac{f(\zeta )d\zeta }{\zeta -i}\Rightarrow \frac{e^{i^{2}a}}{i+i}=\frac{1}{2\pi i}\int_{\Gamma }\frac{e^{ia\zeta }d\zeta }{\zeta ^{2}+1}$

hay $\int_{\Gamma }\frac{e^{ia\zeta }d\zeta }{\zeta ^{2}+1}=\pi e^{-a}$

Ta thấy 

                    $\int_{\Gamma }\frac{e^{ia\zeta }d\zeta }{\zeta ^{2}+1}=\int_{-R}^{R}\frac{e^{ia\zeta }d\zeta }{\zeta ^{2}+1}+\int_{C}\frac{e^{ia\zeta }d\zeta }{\zeta ^{2}+1}$

trong đó $C=\left \{ z,\left | z \right | =R,Imz\geq 0,R> 1 \right \}$

  Ta xét tích phân $\int_{C}\frac{e^{ia\zeta }d\zeta }{\zeta ^{2}+1}$

Đặt $\zeta =Re^{i\theta },\theta \in [0,\pi ]$, ta có

                      $\int_{C}\frac{e^{ia\zeta }d\zeta }{\zeta ^{2}+1}=iR\int_{0}^{\pi }\frac{e^{i\theta }e^{iaR(cos\theta +isin\theta )}d\theta }{R^{2}e^{2i\theta }+1}$.

 Ta có các đánh giá sau

                   $\left | e^{i\theta }e^{iaR(cos\theta +isin\theta )} \right |=e^{-aRsin\theta }\leq 1,\theta \in [0,\pi ]$

              và       $\left | R^{2}e^{2i\theta }+1 \right |\geq R^{2}-1$

nên     $\left | \int_{C}\frac{e^{ia\zeta }d\zeta }{\zeta ^{2}+1} \right |\leq \frac{R}{R^{2}-1}\int_{0}^{\pi }d\theta =\frac{\pi R}{R^{2}-1}\rightarrow 0,R\rightarrow \infty$

    Do đó:

$\int_{-\infty }^{\infty }\frac{e^{iax}dx}{x^{2}+1}=lim_{R\rightarrow \infty }\int_{-R}^{R}\frac{e^{ia\zeta }d\zeta }{\zeta ^{2}+1}=\pi e^{-a}-lim_{R\rightarrow \infty }\int_{C}\frac{e^{ia\zeta }d\zeta }{\zeta ^{2}+1}=\pi e^{-a}$

$\int_{-\infty }^{\infty }\frac{cosaxdx}{x^{2}+1}=Re\left ( \int_{-\infty }^{\infty }\frac{e^{iax}dx}{x^{2}+1} \right )=\pi e^{-a}\Rightarrow \int_{0}^{\infty }\frac{cosaxdx}{x^{2}+1}=\frac{\pi }{2}e^{-a}$